Meyer was found by his employer to have committed scientific fraud by falsifying data. The University of Williamstown...

Lauren on August 22, 2019

“Most Justifies”

@lsatmax I've noticed a couple of questions stems that ask for something the most justifies such as asking for an answer that most justifies the reasoning in the argument or most justifies the decision made is this just another wording for a regular strengthen questions? I'm just a little confused not the best way to approach these problems. Thank you!

Replies
Create a free account to read and take part in forum discussions.

Already have an account? log in

Irina on August 22, 2019

@Lauren-Au,

These questions are known as "principle justify" and they usually require you to identify a general principle that, when combined with the set of facts in the stimulus, makes the conclusion follow logically. The answer choices are all a variation of some sort of a general principle rather than specific facts that you need to apply to the specific circumstances described in the passage and see if the conclusion matches what you would expect if you follow the general principle outlined in the answer choice.

This all might sound confusing, so let's look at this question.

Meyer was found by his employer to have committed scientific fraud by falsifying data. The University of W, from which Meyer held a Ph.D., validated this finding and subsequently investigated whether he had falsified his Ph.D. data and found that he had not. But the university decided to revoke his Ph.D. anyway.

Which principle most justifies the university's decision?

Let's look at the answer choices:

(A) Anyone who holds a Ph.D. from the University of W and is found to have committed academic fraud in the course of pursuing that Ph.D. will have it revoked.

Let's apply this principle to the facts. "anyone who holds a Ph.D. from.." , this is true for M, he holds a Ph.D., "and is found to have committed academic fraud..." Stop. M was found to have committed SCIENTIFIC not academic fraud. (A) principle is inapplicable to this set of facts.

(B) No Ph.D. program at the U of W will admit any applicant who has been determined to have committed any sort of academic fraud.

How is this relevant? M is not applying for admission, he has already completed his Ph.D. and again committed scientific, not academic, fraud. (B) is again inapplicable to this set of facts.

(C) Any U student who is found to have submitted falsified data as academic work will be dismissed from the university.

Irrelevant again. M is not a student, he is a graduate. And he falsified data in the course of his employment, not academic work. (C) is inapplicable to this set of facts.

(D) Anyone who holds a Ph.D. from U of W and is found to have committed scientific fraud will have the Ph.D. revoked.

Let's see. "anyone who holds a Ph.D. from U of W," this is true of M, and "found to have committed scientific fraud," again true of M, "will have the Ph.D. revoked," and U has revoked his Ph.D. as expected. This principle matches the set of facts perfectly and is thus the correct answer choice.

(E) The U of W will not hire anyone who is under investigation for scientific fraud.

Irrelevant. M is not applying for a job at the U and he is not under investigation but was convicted of scientific fraud by his employer already. (E) is inapplicable to the set of facts and is thus wrong.

The justify principle questions are very common on the LSAT and become fairly intuitive with practice. Oftentimes an incorrect answer choice would involve a general principle that is intended to apply to a different set of circumstances than what we have in the stimulus, e.g. student versus alum versus employee, and is thus is inapplicable to the conclusion.

Does that make sense?

Let me know if you have any further questions.

Lauren on August 27, 2019

Thank you so much this helps a lot!!